« first day (41 days earlier)      last day (533 days later) » 
00:00 - 17:0017:00 - 00:00

5:01 PM
0
Q: Simplify these expressions

prishila$$1/(1+tg^2x)+1/(1+cotg^2x)$$ It resultet to me that it is cos^2+sin^2x=1 A I right? The other is: $$tgx/(1-tg^2x)*(cotg^2x-1)/cotgx$$. It result to me that t is sin^2x, but it is not correct

Short title. Short question. Simplify these expressions
0
Q: For each function f defined below determine $f^-1$(C), $f^-1$(D), and $f^-1$ ({1})

bellaLet C={x$\in$R| x$\ge$1} and D={x$\in$R| x$\gt$0}. f(x)=lnx I know that the inverse of this is $f^-1$(x)= $e^x$ so would C be defined as $f^-1$(C)= {x$\in$R| y$\ge$1, x$\in$$e^x$} ?

0
Q: Can't solve the combinatorics question

Serkan KlvzI can't solve the following combinatorics problem Let G=(V,E) be a graph with |V|>=4 and with the property that for any three of its vertices u,v and w,at least two of edges uv,uw and vw are in E.Show that G is hamiltonian.

Words such as question are uninformative in titles. Please edit the title so that it better describes the specifics of your question. Do not hesitate to make it longer or include a formula if needed. More tips here. (autocomment)Normal Human 21 secs ago
0
Q: Existenc of primitive Pythagorean triples

alemonkLet $(a,b,c)$ is a Pythagorean triple, which means $c^2=a^2+b^2$. If $c$ is odd and $a$ & $b$ are relatively prime, then there exist integers $m$ and $n$ such that $c=m^2+n^2, ~a=m^2-n^2, ~b=2mn$. One can easily check the above by proving $\gcd(\frac{a+c}{2},\frac{a-c}{2})=1$. My question is w...

0
Q: How to compute lim (n->-infinity) ((-1)^n n^(1/n) cos(n^n))/(n+1)?

RogetPlease help in computing the limit of ((-1)^n n^(1/n) cos(n^n))/(n+1) as n goes to infinity.

This site uses MathJax formatting of formulas. More tips here. (from a bot)Normal Human 32 secs ago
-2
Q: verify the identity of the two given problems

john315im confused about how to solve this question i managed to solve the ones before this but these two questions seem tricky any help appreciated http://i.stack.imgur.com/KnCrH.jpg

Title contains problem. Short question. verify the identity of the two given problems
 
0
Q: how to find a question once seen realy fast?

Gyro GearlooseI'm having problems with my internet connection. I saw a question which I identified as a duplicate of a question of my own, but my connection fainted, I had to restart the browser and the link was lost. I remember it was from February 2015, tagged jsf or jsf-2, had to do with ajax and keyboard...

 
0
Q: Show f(x) < x*f'(x)

JulesFunction f:[0,inf)->R continuous on [0,inf) and differentiable at (0,inf). f(0)=0. f' is strictly increasing.

Short title. Short question. Show f(x) < x*f'(x)
0
Q: Polynomial Formula for Series

user2620316So far, I'm stuck on this problem of converting a series to a polynomial and showing that it exhibits certain properties. PROBLEM Show that the polynomial formula for $P_k(n) = \sum_{j=1}^n j^k$ is characterized by the following two properties: $P_k(0) = 0$ for all $k$ $P_k(x)-P_k(...

0
Q: $g \in H$ - Abstract Algebra

J.GLet $G = A_5$ and $H=\langle(1,2,3,4,5)\rangle$. Let $g \in N_G(H)$ an element of order $5$. Compute the order of $H \langle g \rangle$ (already done) and $g \in H$. I already know that both of the subgroups $H$ and $<g>$ are of order 5 which is prime, so either they are equal, or their intersec...

0
Q: compact set in $\mathbb{R}$?

kawazakii am studing the (Schrodinger operator) but this question is in topology: i found in a course that this set : $DS_\alpha(\lambda,\gamma)=\{\beta\in\mathbb{R}| \forall k\in\mathbb{Z}-\{0\},\forall l\in\mathbb{Z} : |\beta-k\alpha-l|\geq\frac{\lambda^{-1}}{|k|^\gamma}\}$ thank you very much

0
Q: Did I simplify these expressions correctly?

prishilaI am writing the expression and my final result. If there is any other step to add or if I have done any mistake please correct. cotgx-cotg2x=2/sinxcosx sin2x/(1+cos2x)*cosx/(1+cosx)=sinx/(x+cosx) (sinx+sin3x+sin5x)/(cosx+cos3x+cos5x)=tg3x

This site uses MathJax formatting of formulas. More tips here. (autocomment)Normal Human 20 secs ago
0
Q: Tangent identity

JohnI came across this in a paper, and although I suspect its extremely elementary, I can't quite grasp it. Suppose we have vectors $\vec{x}$ and $\vec{x}^{\prime}$. As a given, we have \begin{equation} \tan\left[2\Theta(\vec{x}-\vec{x}^{\prime})\right] = \frac{2(x_{1}-x_{1}^{\prime})\cdot(x_{2}-x_...

Short title. Tangent identity
0
Q: Show that given seven real numbers always is possible take two of them, such that $\left\vert\dfrac{a-b}{1+ab}\right\vert<\dfrac{1}{\sqrt{3}}$

pablocn_ Show that given seven real numbers always is possible take two of them, such that $$\left\vert\dfrac{a-b}{1+ab}\right\vert<\dfrac{1}{\sqrt{3}}$$ The "Pigeonhole principle" states that if $n$ items are put into $m$ containers, with $n > m$, then at least one container must contain more than o...

0
Q: Show Bessel's inequality

Joe Mahanasha$x_{1}, ..., x_{k}$ is orthonormal. Show that for any $y \in X$: $\sum_{i=1}^{k} \left | \left \langle x_{i}, y \right \rangle \right | ^{2} \leqslant \left \|y \right \|^{2}$

Short title. Short question. Show Bessel's inequality
Tall formulas in titles break the layout of question lists. Please replace \dfrac with \frac in the title. (autocomment)Normal Human 20 secs ago
 
5:28 PM
-1
Q: Require comment when voting down

lol5433How about require a comment when voting down a question? The user will then understand why the question was bad and instead not confused by it.

 
0
Q: Equations in fields.

Théodore Rozencwajg So a) is simple, calculate the squares and cubes mod 7. However, for b) I know that Z7[x]/(f(x)) if f(x) is irreducible of degree 2 would work, but then how do I solve the equations ? I assume F7xF7 would also contain 7^2 elements but is it possible to solve the equations in that field ? How ...

Short title. Equations in fields.
0
Q: Taxi - A stochastic Process

EffConsider the following problem. A small taxi company has two taxis that each have been distributed to separate halves of a city. If a person orders a taxi, then the taxi that is distributed to that part of the city will come; unless that taxi is already occupied and the other is free, then the ot...

0
Q: Problem on conditions on the divisors of integers

SigmaI've recently started exploring elementary number theory, and came across the book Number Theory for Beginners by André Weil, which is where I found this problem. The problem is: Prove that any integer $x>1$ has either a divisor $>1$ and $\leq > \sqrt{x}$ or no divisor $>1$ and $<x$ `` I'm...

0
Q: prove that lim is 1

Shachar Orenlim\frac{1}{n}\left ( \frac{\sqrt{2}}{\sqrt{1}} \right )\left ( \frac{\sqrt{3}}{\sqrt{2}} \right )\left ( \frac{\sqrt{4}}{\sqrt{3}} \right )...\left ( \frac{\sqrt{n+1}}{\sqrt{n}} \right )= 1 I know that ( \frac{\sqrt{2}}{\sqrt{1}} \right )\left ( \frac{\sqrt{3}}{\sqrt{2}} \right )\left ( \frac{\...

This site uses MathJax formatting of formulas. More tips here. (autocomment)Normal Human 20 secs ago
0
Q: Showing a solution is unique

user118224 Let $F:\;[a,b]\times \mathbb{R}^n\to\mathbb{R}^n$ where $0\in [a,b]$ and for some $K>0$: $$||F(t,x)-F(t,y)||\leq K||x-y||$$ for all $x,y$ and all $t\in [a,b]$. I would like to show that there is unique continuous $f:[a,b]\to\mathbb{R}^n$ solving: $$f(t)=\int_{0}^t F(s,f(s))\; ds$$ ...

0
Q: A real number exponentiated to a real number is positive

lorenzoLet $x>0$ and $q$ be real numbers. Show that $x^q$ is a positive real. DEF.(Exponentiation to a real exponent) Let $x>0$ be real, and let $\alpha$ be a real number. We define the quantity $x^{\alpha}$ by the formula $x^{\alpha}=lim_{n\to\infty}x^{q_n}$ where $(q_n)_{n=1}^\infty$ is any sequence ...

Please don't use (self-learning) tag just because you were self-studying. This tag is only for questions about the process of self-studying. (from a bot)Normal Human 20 secs ago
0
Q: Orbifolds in dynamical systems theory

0537Is there any use of the concept of an orbifold in dynamical systems theory? Can orbifolds be applied to any problems in dynamical systems?

0
Q: finding A of a matrix representation of a linear transformation

user3533755How do you find A in a matrix representation in the linear transformation $[Lv]_E = A[v]_E$

0
Q: Solving Sakuma Hattori equation with MATLAB

G StieglerThe equation recommended to model the signal measured by a radiometer is the Planck form of the Sakuma–Hattori equation $S(T) = \displaystyle\frac{C}{\exp\left(\frac{c_{2}}{AT+B}\right)-1}$ where $c_{2}$ is the second radiation constant, and $A$, $B$ and $C$ are related to the radiometer’s spec...

0
Q: find a function such that

user298411I'm having some trouble knowing where to start with this problem. Find an example of a function such that $$f(x)\neq-2x$$ such that $$\int_{0}^{1} \left[ -2x-f(x) \right] = 0$$ I'm looking for a nudge in the right direction rather than a complete solution.

0
Q: Specific confidence intervals question

user5675285As of now, I completely suck at computing confidence intervals, and I have an exam in a couple of days. The exact question is as follows: Some people are trying to measure a certain parameter $\theta$. Every measurement has an error, and this error has the normal distribution with mean 0 and st...

Welcome to Math.SE, user5675285. Words such as question are uninformative in titles. Please edit the title so that it better describes the specifics of your question. Do not hesitate to make it longer or include a formula if needed. More tips here. (from a bot)Normal Human 22 secs ago
0
Q: Minimum guard problem

MahmoudPlacing 2x2 dominoes on an 8x8 chess board, in non-overlapping way, what is the lowest possible number of dominoes to lock (guard) the board, so that no further dominoes can be placed on the board? The aim here is to cover as little as possible and save the maximum number of unused dominoes Than...

Questions tend to get more attention when they have a tag for a broad area of mathematics relevant to the question. Some of these tags might fit. (from a bot)Normal Human 30 secs ago
0
Q: Prove that if the complex function $|f(z)|^2$ is constant in $D$ and $f(z)$ is analytic in $D$, then $f(z)$ is constant in $D$.

PatternsMy proof: Let $|f(z)|^2 = M$ for $z\in D$. Then $f(z) = \pm\sqrt{M}$ (not sure about this step, are there only two values for the square root of a complex number> No right? Could be more. But I don't think it would change the essence of the proof) But $f(z)$ is analytic in $D$ so it cannot be...

0
Q: Algebraic Topology by Rotman Exercise 6.8

EnigmaSo I've been working through some of the suggested exercises through Rotman and I have one problem that took longer than I expected. Most of the starred exercises seem to have a short quick proof except this one,I was wondering if someone could help me see the faster way to do this one. Problem:...

0
Q: Unambiguous solution

User 11111For $a \in R$ we got matrix: $A = \begin{bmatrix} a & 1 & 1 &0 \\ 1 & a & 1 & 0\\ 1 & 1 & a & 0\\ 1 & 1 & 1 & 1 \end{bmatrix}$. For which $a \in R$ equation $A\overrightarrow{x} = \overrightarrow{b}$ got unambiguous solution ($\overrightarrow{x} \in R^{4}$) for every vector $\overrightarrow{b...

Short title. Unambiguous solution
 
6:17 PM
0
Q: What's the distribution?

Sung Pang$$X_1,X_2,...,X_n \;i.i.d\;with\,density\;function$$ $$f(x)=e^{-(x-\theta)}\;,x\ge\theta$$ What is the distribution of $\bar X$? I gained m.g.f of $\bar X$. (by calculating $E[e^{{1/n} (X_1+...+X_n)}])$ Result is $\frac n {n-t}e^{\frac {t\theta} n}$. But I don't know distribution that has th...

0
Q: Am I missing something in this question or is it really one-line? (Fubini or not)

Hawk If $h(x,y) = f(y)g(x)$ and $f$ is a function on $(X, \mu)$ and $g $ on $(Y, \nu)$, then $\int_{X \times Y} h d(\mu \times \nu) = \int_Y g d\nu \int_X f d\mu$ and $h$ is $\mu \times \nu $ integrable. Isn't the whole thing just $$\int_{X \times Y} h(x,y) d(\mu \times \nu) = \int_{X \times Y}...

0
Q: Consider the space MxN

user298418Let (M,d) and (N,p) be metric spaces. Consider the space MxN endowed with the metric D=((x,y),(x',y'))=max{d(x,x'),p(y,y')} for (x,y),(x',y') in MxN. Let A in M and B in N be nonempty. Prove or disprove: a) P:MxN ->M, P(x,y)=x is continuous and an open mapping b) The set MxN is totally bounde...

Welcome to Math.SE, user298418. Consider replacing (analysis) with a more specific tag for the relevant branch of analysis. (from a bot)Normal Human 21 secs ago
0
Q: trigonometry in calculus intervals

user122415 So the original function was f(x) = cotx + x Now I thought -csc^2(pi/6) = -4. So -4+1 = -3 hence it should be negative from the interval (0,pi/2). Somehow I have the feeling I don't even know high school trig... please help me out my brain is exploding right now i can't think straight.

This site uses MathJax formatting of formulas. More tips here. (from a bot)Normal Human 21 secs ago
0
Q: Is the exponential map to the indefinite special orthogonal groups $SO^+(p,q)$ surjective?

FriedrichIs the exponential map to the identity component of the special indefinite orthogonal groups $$ \mathrm{exp} \colon so(p,q) \to SO^+(p,q)$$ surjective?

0
Q: The relationship between sample variance and proportion variance?

Stella ParkI'm trying to see the relationship between the sample variance equation Sigma(Xi-Xbar)/(n-1) and the variance estimate, Xbar(1-Xbar), in case of binary samples. I wonder if the outputs are the same, or if not, what is the relationship between the two?? I'm trying to prove their relationship ...

0
Q: What is 100,000 as a sum of squares?

user298365I cannot find the solution to this problem. It is part of a larger homework question but I can't go on until I solve this question.

Consider adding a tag for a broader subject area to which the question belongs. Some of these tags might fit. (from a bot)Normal Human 23 secs ago
0
Q: simple RSA decypher

WatsonI think my confusion here is just which how the question was given to me. I am having trouble decrypting this simple RSA message. Message: 0882 1090 1471 1899 2753 0309 p = 43 ; q = 71; e = 19. Someone please check my work that I have so far and I would appreciate help getting the final answer...

Short title. simple RSA decypher
0
Q: density and inequality

user298344I understand $C^{\infty}_0 (\mathbb{R}^3)$ is dense in $H^1 (\mathbb{R}^3)$. But I don't understand the reason it is enough to proof the following Hardy inequality if you proof the case $u \in C^{\infty}_0$. (Hardy inequality) Let$ \ u \in H^1 (\mathbb{R}^3)$. Then, $$\int_{\mathbb{R}^3} \frac{...

 
6:54 PM
0
Q: Proving that a closed plane in R3 has a point in the plane closest to an arbitrary point outside the plane

user298418Let A={x in R^3: |x_1| + 2|x_2| +|x_3|^3 = 1 } and let p in R^3/A ..Show that there exists a point y in A that is closest to P among all points in A. ** Assume R^3 has the euclidean metric I obviously want to do this by contradiction, but I'm unsure of what assuming there is no single point clos...

Welcome to Math.SE, user298418. This site uses MathJax formatting of formulas. More tips here. (from a bot)Normal Human 25 secs ago
0
Q: Rank of some special matrices

TurboIs every $\{-1,a,1\}^{n\times n}$ matrix with $a$ occuring only on the diagonal full rank? Only condition on $a$ is that it is not in $[-1,1]$. What if $a=0$? What if $a=2$?

 
7:10 PM
0
Q: Show that if A, B, C, D are sets, |A| = |B| and |C| = |D| then |A × C| = |B × D|.

Ramsey BissexI know that if A and C are finite sets then |AxC|=|A||C|. This makes the problem quite simple but the sets may not be finite. I am guessing that the concept of cardinally of infinite sets and ℵ 0 are part of the solution but those are concepts that my class did not go into much and I do not und...

0
Q: Cycles of length 4 and paths of length 2 Km,m

HasLet say we have $m,n \in \mathbb{Z}\ with\ m \geqslant n \geqslant 2.$ How many cycles of length equal to 4 are in $k_{m,n}?$ And how many paths of length equal to 2?

0
Q: Predicate Logic Representation

zimHow do you represent arithmetic operations with predicate logic? For example if I want to represent 2+3 = 5 in predicate logic with plus(x,y) function is the following correct? plus(2,3) <=> 5 But ideally <=> is equivalent and not equal. So is the above representation wrong?How do you represent...

0
Q: On an exercise on restrictions of functions in a topological space.

MonoliteI rewrite the text of an exercise: Given $X$ and $Y$ topological spaces and a family of subsets $\{A_i \}_{i \in I}$ of $X$ s.t. $X = \cup_{i \in I} A_i$ . Let $\{ f_i : A_i \rightarrow Y \}_{i \in I}$ be a family of continous function such that $f_{i|A_i \cap A_j} = f_{j|A_i \cap A_j}$ then ...

 
7:26 PM
0
Q: Tag merge / synonym request: [ux] → [user-experience]

gnatPlease merge tag ux (13 questions) into user-experience (68), and make them synonyms. Neither of the tags has wiki, but questions in both are about user experience (UX). Preference for master tag is because it has much more questions.

 
0
Q: Counting problem involving permutations - verification please?

ceeksI have tried solving this, but I'm unsure if I'm right. Any suggestion would be appreciated. Question: 15 kids arrive at camp and are assigned a place to sleep. There are 3 different cabins each of which can hold 5 kids. How many ways are there to assign kids to cabins? My answer: (15,5) (mean...

0
Q: Strictly ergodic preserving order maps on finite set

BelochkaLet $X = {1, 2, ..., 10}$ How many strictly ergodic, preserving order maps are on this set?

0
Q: How to solve a system of equations with sin and cos?

lindsey laurune50= 35cos(x) +25cos(y) 0= 35sin(x)+25sin(y) Thanks!

0
Q: Suppose that A ⊆[0,1] is measurable set such that m(I∩A)≤m(I)/2, for all intervals I⊆[0,1]. Show that m(A)=0 .

Amer DradkaSuppose that A ⊆[0,1] is measurable set such that m(I∩A)≤m(I)/2, for all intervals I⊆[0,1]. Show that m(A)=0 .

Short question. [Suppose that A ⊆[0,1] is measurable set such that m(I∩A)≤m(I)/2, for all intervals I⊆[0,1]. Show that m(A)=0 .](math.stackexchange.com/q/1574046)
0
Q: Using change of variable to solve a partial differential equation

ChristianI've been practicing on PDEs, since I haven't needed to work with them in a long time until now, when I got stumped on how to solve this problem. The problem asks to use the change of variable $$u(x,t) = w(x,t) + q(x)$$ to solve the PDE $$ \frac{\partial u}{\partial t} = 3\frac{\partial^2 u}{\...

Tagged pde, differential-equations. Tagged differential-equations but mentions "partial". Using change of variable to solve a partial differential equation
0
Q: Question about schur's lemma and irreducible representations of S_n

010110111Schur's lemma says that if $M,N$ are two irreducible representations of a group $G$, then either $Hom_G(M,N)=0$ if $M,N$ are not isomorphic, or every $\varphi\in Hom_G(M,N) $ is invertible if they are isomorphic. If we look at the case $G=S_n$ and $M=V_\lambda,\ N=V_\mu$ for partitions $\lambda,...

0
Q: having trouble with this problem ( dont understand what to even do)

user298436Consider the following recursive definition of a function: g:NN 1) Base Case: g(0)=0, 2) Recursive Case: For any x>0 we have g(x) = g(x-1) + 2 Prove each of the following properties holds for g using induction on nEN. i) g(n) = n+n;

Title contains troubl, problem. Tagged proof-theory. having trouble with this problem ( dont understand what to even do)
0
Q: Test of convergence

Mathematics Can someone help me with this particular question? I don't know how to proceed.

Short title. Short question. Test of convergence
0
Q: Apollonius' Problems

user userI am trying to solve all Apollonius' Problems.i solved 7 of them.but i can't solve the remains (PCC-LLC-CCL). i checked some pages like :(http://www.physics.princeton.edu/~mcdonald/papers/apollonius_051964.pdf) or(https://en.wikipedia.org/wiki/Special_cases_of_Apollonius%27_problem). and etc ,but...

Short title. Title contains problem. Question contains step by step. Apollonius' Problems
0
Q: Proving that $O(2^{f(x)})=2^{O(f(x))}$

user216094How can I prove above equality using the definition of Big O? $O(2^{f(x)})=2^{O(f(x))}$

Short title. Short question. Proving that $O(2^{f(x)})=2^{O(f(x))}$
0
Q: When echelon form equals the init matrix: Silly problem

darkmoorI am trying to solve the following exercise: Describe alla $2$ by $3$ matrices $A_1$ and $A_2$ with row echelon forms $R_1$ and $R_2$, such that $R_1 + R_2$ is the row echelon form of $A_1 + A_2$. Is it true that $R_1=A_1$ and $R_2=A_2$ in this case? According to the exercise: $$\left. \be...

0
Q: A question about a case where Central Limit Theorem doesn't apply

AnlamKI'm trying to read Lehmann's "Elements of Large Sample Theory" and I have the following question about the text. The classical Central Limit Theorem is stated as: Now, the author goes on to provide the following counter-example: My question is, what assumption of the CLT does this counter-...

0
Q: I dont understand

Help plzLimit. Sqrt (x^2+x) - sqrt (x^2-x) (x -> infinity) I get 2 answers for this question 1 and 0 but 1 is the right answer. I dont know why it is like tht though. If u multiply by the conjugate ÷ conjugate (1) u take the sqrt out of top and get it in the bottom and then if u factor out x from ...

Welcome to Math.SE, Help plz. This site uses MathJax formatting of formulas. More tips here. (autocomment)Normal Human 21 secs ago
0
Q: Rotation Matrix from i to Arbitrary Unit Vector

The Trombone WillyThe title just about explains everything. What is the 4x4 rotation matrix from i or (1.0, 0.0, 0.0) to an arbitrary unit vector, such as (0.424, 0.565, 0.707)?

0
Q: n where it gives certain remainder for certain number

Sohni MahiwalI am studying for GRE and need help with following question When the positive integer n is divided by 3, the remainder is 2 and when n is > > divided by 5, the remainder is 1. What is the least possible value of n? Answer says 11, but gives no explanation. I tried to solve this, i.e. n = ...

This site uses MathJax formatting of formulas. More tips here. (from a bot)Normal Human 21 secs ago
 
8:23 PM
0
Q: Editing the problem by an answerer to "make" their answer "correct"

MirkoI just noticed that user @EDUARDO has edited the problem posted by @Ashok Vardhan, apparently with the aim of adding an extra hypothesis (namely that the measure of the derived set of the $r_n$ is $0$) that would "make" the incorrect answer by @EDUARDO "correct". (I had left earlier comments to @...

 
0
Q: How to calculate this force

JacksonFitzsimmonsHow can I calculate the force exerted on the sides of a trough when the trough is filled with water, in general? Meaning for a given shape and depth of water, how can i calculate the force exerted on a given side of the shape?

0
Q: Solvable group - Simples factors - decomposition sequence of $G$ - Abelian groups

J.GShow that a finite group $G$ is solvable group (in the sense there exists an $n$ such that $G^{(n)}=1$) if and only the simples factors in a decomposition sequence of $G$ are all abelians. I'm not able to solve this problem. Is anyone is able to give me a hint?

Questions tend to get more attention when they have a tag for a broad area of mathematics relevant to the question. Some of these tags might fit. (autocomment)Normal Human 21 secs ago
0
Q: Should I take courses that I self-studied in High School again in University?

cheeseinthetrapI'm a grade 12 student in Canada. I studied Multivariable Calculus (Larson), Linear Algebra (Strang), Proving (Velleman), Abstract Algebra (Fraleigh) and Differential Equations (Simmons & Krantz) and I'm currently studying Probability and Statistics (Tsitsiklis). None of these courses are too r...

0
Q: Take an integral of tan x and sec x to a power of something without using reduction formula?

Abdulaziz AsiriAssume p is a positive integer > 1. can I take the integral of tan^(p)*x without using reduction formula ? The same applied to sec^(p)*x Thank you.

This site uses MathJax formatting of formulas. More tips here. (from a bot)Normal Human 21 secs ago
0
Q: help understanding definition of pullback and calculating it

mathematics2let $w = A(x,y,z)dy\wedge dz$ where $y = h(x,z)$ and $(x,z) \in D = [0,1]^2$. Let $c$ be the singular 2-cube on A, i.e. $c: [0,1]^2 \to A$ is continuous. I am trying to calculate the pullback $c^{\star}w$ from definition of pullback I have that $c^\star(A) dy \wedge dz = A(c)(c^\star(dy \wedge ...

 
8:57 PM
0
Q: Central limit problem. Calculate the probability that the flight will be overbooked.

Electro82The problem said: An airplane has 120 seats. The probability that a ticketed passenger will show up for a flight is 0.95. Assume that all passengers act independently and that the airline has sold 130 tickets for a particular flight. Using the Normal approximation to the Binomial (wi...

0
Q: For what values of a and b does the system have a unique solution/many solutions or no solution?

Roy ArwaLet $$ \left[ \begin{array}{ccc|c} a & 0 & b & 2\\ a & a & 4 & 4\\ 0 & a & 2 & b\\ \end{array} \right] $$ be the augmented matrix for a linear system. For what values of a and b does the system have: a) A unique solution; b) Infinitely many solutions; c) No solutions. Could you plea...

0
Q: Laurent series $\frac{1}{e^z-1}$

AmithHow can I expand $$f(z)=\frac{1}{e^z-1}$$ into Laurent series? I know that $f$ has singularities in $2k \pi i, \ \ k \in \mathbb{Z}$. Just substituting Taylor series for $e^z$ in the denominator doesn't give me much.

 
0
Q: a tumblr blog for theme-related stackoverflow questions

S Lewisi created this tumblr blog to answer a question that a user had regarding photo posts. since we can't simulate the site on here, perhaps we can add more members to it and all use this blog to showcase examples for tumblr theme-related questions? we can create example posts for things like webkit...

 
0
Q: Christmas problem, the salesman with the nuts

algebra1At the Christmas market, a man was selling nuts in a market stall. The first person bought one nut, the next customer bought two nuts, the next bought four, and so on. That is, every new customer acquired twice as many nuts as the previous one. Last customer of the day bought 50 kg of nuts,...

Tag (problem-solving) should not be the only tag a question has. Please add a tag for a subject area to which the question belongs. (autocomment)Normal Human 21 secs ago
0
Q: $L^\infty$ is complete proof from exercises in Royden & Fitzpatrick's Real Analysis.

CraigI've been reading up on some Analysis for my comp exams, and I couldn't find in my texts a proof of $L^\infty$ being Banach. Someone pointed me to the following exercise in Royden & Fitzpatrick. Now, I've found other proofs of $L^\infty$ being Banach online, but this problem is now really botheri...

0
Q: Herschel-Maxwell derivation

Sagke foondI have a question about the Herschel-Maxwell derivation, as described in the text here: http://www-biba.inrialpes.fr/Jaynes/cc07s.pdf Specifically, equation (7-4). I understand why the probability should not care about the angle $\theta$, that is I understand that the integral over the density...

0
Q: Basis for null space

gbox find the basis of the null space \begin{pmatrix} 16 & 32 & -40 & 24 \\ -3 & -6 & 7 & -4 \\ 80 & 160 & -176 & 96 \end{pmatrix} the row reduced echelon form is: \begin{pmatrix} 1 & 2 & 0 & -1 \\ 0 & 0 & 1 & -1 \\ 0 & 0 & 0 & 0 \end{pmatrix} $x+2y-s=0\righta...

Short title. Basis for null space
 
9:32 PM
0
Q: American Statistical Association Government Statistics Section Data Challenge

StasKThis is an announcement rather than a discussion entry. Stuff like that is definitely off-topic on the main site, and while I have reservations regarding the Meta, either, I'd still like to post this opportunity. The Government Statistics Section of ASA is sponsoring Data Challenge 2016 to take ...

 
0
Q: Can sets have derivatives?

Guacho PerezI know how functions can be described, e.g. $y=x^2$ and in high school they teach you the general form, e.g. $x^2-y=0$ I believe so that later on they can abstract the notion of the curve into a set-theoretic notion, e.g. $S=\{ (x,y) : x^2-y=0\}$ and then the graph is just the same as coloring th...

0
Q: How to calculate such an integral?

marmistrzI have this integral $$\int \frac{\sqrt{x}}{\sqrt{1-x}}dx$$ I tried integrating it with integration by parts, using $u = \sqrt t$, trigonometric substitutions, but I'm stuck. Can you help me please?

Question contains please. How to calculate such an integral?
0
Q: Limits and rate of change

daniel furhangI'm a freshman taking calculus 1 currently studying for finals. I am reviewing stuff from the beginning of the semester,and I don't remember the proper way to deal with limits like this one. A ball dropped from a state of rest at time t=0 travels a distance $$s(t)=4.9t^2$$ in 't' seconds. I am t...

0
Q: units of group ring $\mathbb{Q}(G)$ when $G$ is infinite and cyclic

user0990How would I be able to describe all units of the group ring $\mathbb{Q}(G)$ where $G$ is specifically an infinite cyclic group?

0
Q: Eggs and Cranes, What is the probability that it is a whooping crane’s nest?

DarioCThe problem said: In a certain region, blue cranes are twice as common as whooping cranes. Suppose that the number of eggs laid by a blue crane is a Poisson(! = 3) random variable and the number of eggs laid by whooping crane is a Poisson(! = 5) random variable. You find a crane’s nest ...

This site uses MathJax formatting of formulas. More tips here. (autocomment)Normal Human 21 secs ago
1
Q: Evaluating Indefinite Integral

user97624∫ 1/((1-x^3)^(1/3)) dx I tried substituting 1-x^3 as t^3 but I am not able to calculate it after that. Thanks!

This site uses MathJax formatting of formulas. More tips here. (autocomment)Normal Human 21 secs ago
0
Q: Problem solving: How far is the maximum distance?

algebra1The tires located on the front of the car wears out after 25000 km, while the tires on the back wears out after 15000 km. How far can you maximum ride with new tires if you can swap the tires during the journey?

0
Q: How do I prove that the unit speed curve is a helix?

A. UserHow do I prove that the unit speed curve is a helix?

Welcome to Math.SE, A. User. Questions tend to get more attention when they have a tag for a broad area of mathematics relevant to the question. Some of these tags might fit. (from a bot)Normal Human 21 secs ago
0
Q: Can this integral be solved and obtained in a closed form?

HamidI have this integral that I need to solve: $\int_a^b \sum_{k=1}^K \max(x,a_k) \text{d}x$ for some constant $a_k\in\mathbb{R}, k=\{1,\dots,L\}$.

0
Q: Surjective quadratic mapping

Leo EulerAre there any know values of $n$ for which there exists a surjective quadratic mapping $Q:\mathbb{R}^n \rightarrow \mathbb{R}^n$ with non-trivial zeroes?

Short title. Short question. Surjective quadratic mapping
0
Q: confusion with an excersise of proyective plane

Iván García BraoI have a problem with this exercise, I have been thinking about to use some propierties of proyective plane and conics, but I dont know how to solve it. can you help me please? the exercise is: "proof that if a triangle is circumscribed to a conic projective nondegenerate then the lines connecti...

Title contains confus. Question contains please. confusion with an excersise of proyective plane
0
Q: every non-zero prime ideal of $K[x,y]/(f)$ is maximal ideal

corcia candyLet $R=K[x,y]/(f)$ where $f(x,y)=y^2-x^3$ I can show R is integral domain and Noetherian. But I have to show every non-zero prime ideal of $R$ is maximal ideal. but I can not realize form of ideal of $R$ I try to choose a prime ideal $P$ and want to show $R/P$ is (finite integral domain so is f...

Question contains please. [every non-zero prime ideal of $K[x,y]/(f)$ is maximal ideal](math.stackexchange.com/q/1574226)
0
Q: Define a function map

user390293Given $\sigma : G → (G/M) $ x $G/N$. Define $\sigma$. Can someone please check if I have defined it well. $\sigma(g) = ( g + M, g+ N)$ or $\sigma(g) = ( gM, gN)$ ? would either work given $M,N$ are normal .

Welcome to Math.SE, user390293. Consider adding a tag for a broader subject area to which the question belongs. Some of these tags might fit. (from a bot)Normal Human 21 secs ago
1
Q: 3 Urns, dimes and pennies. If two of the three coins are dimes. What is the probability that the coin selected from urn I was a dime?

JulietaRProblem said: Urns I, II and II contain three pennies and four dimes, two pennies and five dimes and three pennies and one dime, respectively. One coin is selected at random from each urn. (a) What is the probability that all three selected coins have the same denomination? P(a:s...

This site uses MathJax formatting of formulas. More tips here. (autocomment)Normal Human 21 secs ago
 
10:12 PM
0
Q: Question accepted - rep delay possible?

ThisClarkIs there occasionally a delay in rep updates? My rep log shows a +2/-2 which should cancel out, but my net total is down 2. I check the rep log in profile and it is inconsistent with the above info. The +2 is not updated. Might there be other factors at play, such as pending flag for duplicate...

 
0
Q: How to know if a sum is less than a given number

user298467It is given that $\frac{1}{2^j}>\frac{1}{2^{j+1}}$. So is it true that $\sum\limits_{n=j+1}^\infty \frac{1}{2^i}<\frac{1}{2^j}$?

0
Q: Question regarding constructing a function via its Fourier transform

Johnny T.Let $\varepsilon>0$. I was interested in understanding the justification of defining the following function $\phi$ via its Fourier transform, satisfying the following properties: (1) $\widehat{\phi}\in C^\infty(\mathbb{R})$ (2) $\widehat{\phi}(\xi)=1$ for $\xi\in[-\pi+\varepsilon,\pi-\varepsil...

0
Q: Is this sum zero?

The HomeworkerLet $\{y,...,x\}$ be a finite subset of the natural numbers. Does this imply that $$\sum_{l=y}^{x} \prod_{ p \in \{y,...,x\} \backslash\{q\}} \prod_{q \in \{y,...,x\} \backslash \{l\}} (\delta_{q}-\delta_p)=0$$ for an arbitrary sequence of numbers $(\delta_n)_{n \in \mathbb{N}}.$ I have the f...

Short title. Is this sum zero?
 
0
Q: Audit review does not give me any options

JakujeReview audit gave me answer, without any possibility to do anything else than "No action needed" or "Skip" (see attached image). There is no way to vote, no way to comment, no way to flag. The reason is probably that the question is locked. Is it expected, that we should Skip such answers, ev...

 
10:34 PM
0
Q: Poisson process on random time interval

user230329Let $N(T) \sim P(\lambda T)$, and $T\sim U[0,1]$. Compute $E(N^\alpha(T)|T=t)), \, E(N(T))$, where $\alpha \in \mathbb{N}$. Clearly that $\{N(T)|T=t\} \sim P(\lambda t)$, hence $E(N^\alpha(T)|T=t)$ is the $\alpha$'s moment of $P(\lambda t)$. For the second case, \begin{align} E(N(T)) &= E\lef...

Tagged proof-verification. Poisson process on random time interval
0
Q: Why was this terminology of holomorphism used here?

PhilosophicalPhysicspage 6 in this notesays: A note on index convention: $i$,$j$, and $\bar{i}$, $\bar{j}$ index over the holomorphic and antiholomorphic components. I never knew there were holomorphic components, I only was aware of holomorphic mappings and holomorphic functions. Can someone please point ou...

0
Q: Factoring Multivariable polynomials

user298469Determine a constant $k$ such that the polynomial $$ P(x, y, z) = x^5 + y^5 + z^5 + k(x^3+y^3+z^3)(x^2+y^2+z^2) $$ is divisible by $x+y+z$. In this multi-part problem, we will consider this system of simultaneous equations: 3x+5y−6z5xy−10yz−6xzxyz===2,−41,6.(i)(ii)(iii) 3x+5y−6z=2,(i)5xy−10yz−6x...

0
Q: Example of signed measure negative

MSEDo you have an example of signed measure that take negative value ? I didn't found any example. By the way, do you have an explanation of those signed measure ? I don't see any utility.

0
Q: Inverse sum headache

Henning ScheiI'm now extreamly tierd of not pulling off this equation. $\sum_{i=1}^n (y_i-\alpha)^2= \frac{2n\sum_{i=1}^n (y_i - \alpha)}{\sum_{i=1}^n (\frac{1}{y_i - \alpha})}$ Solve for $\alpha$, y is a stochastic variable.

Short title. Inverse sum headache
0
Q: prove that a function from a power set to the set of all functions between two sets is bijective

salvador lazaroI am reading Amann's book of Analysis I and I am trying to prove the following: For each non empty set X , the function P(X)↦{0,1}^x,A↦X_A is bijective Where P(X) according to the Book is the power set of X , X_A is the characteristic function of A defined: X_A:X↦{0,1}^x ,x↦1 if x∈A or x↦0 if ...

0
Q: Induction Proof with Fibonnaci

Eugene WalesHow do I prove this? For the Fibonnaci numbers defined by $f_1=1$, $f_2=1$, and $f_n = f_{n-1} + f_{n-2}$ for $n$ ≥ 3, prove that $f^2_{n+1} - f_{n+1}f_n - f^2_n = (-1)^n$ for all $n$ ≥ 1.

This site uses MathJax formatting of formulas. Consider replacing (analysis) with a more specific tag for the relevant branch of analysis. More tips here. (from a bot)Normal Human 21 secs ago
0
Q: Please verify this proposition on smallest grammar computation.

Enjoys MathLet $G$ be a smallest grammar for a string $s$ that has two terminal rules: $A = abc; \ B = bcd$ Add the rule $C = bc$ to the grammar and also change the rules to $A = aC; \ B = Cd$. Then we still have a minimal grammar for $s$, $G'$. Clearly given $G'$ we can compute $G$ in $O(k)$ time where...

Title contains please. Tagged proof-verification. Please verify this proposition on smallest grammar computation.
 
11:01 PM
0
Q: Zero answers but accepted is "yes"

BSMPBelow is a screenshot of an audit in Triage. If you look at the circled info in the right hand column, you'll see that the audit claims that the question has zero questions but has accepted one. When you pass this audit all the other info on the right hand side changes: Is this a bug? It se...

 
0
Q: need help with solution for this induction problem

user298481Consider the following recursive definition of a function: g:N→Ng:N→N. 1) Base Case: g(0)=0g(0)=0, 2) Recursive Case: For any x>0x>0 we have g(x)=g(x−1)+2g(x)=g(x−1)+2. Prove each of the following properties holds for gg using induction on n∈Nn∈N. i) g(2*n) = 2*g(n)

0
Q: Why this solution works?

ZANGiven two integers: n and m and n is divisible by 2m, I have to write down the first n natural numbers in the following form. At first first m integers are taken and their sign negative is made negative, then next m integers are taken and their sign is made positive, the next m integers should...

Welcome to Math.SE, user298481. Words such as help are uninformative in titles. Please edit the title so that it better describes the specifics of your question. Do not hesitate to make it longer or include a formula if needed. More tips here. (from a bot)Normal Human 24 secs ago
This site uses MathJax formatting of formulas. More tips here. (from a bot)Normal Human 21 secs ago
0
Q: Reflections of Congruent Rectangles

samsoniteIs there a way in which we can guarantee 3 reflections or fewer will map any two congruent rectangles (ABCD and WXYZ) to each other with?

0
Q: Solving equations in fields.

Théodore RozencwajgHow does one solve x²-3 and x²+x-1 in $F$7[x] / (x²-5) ? Not sure how do do it, I know there is a solution for both, but I don't see how to get there. Thanks.

0
Q: How to prove this problem ($g(x)$ approximated by nondecreasing $f_n(x)$)

sleeve chenHow to show the following problem: My approach is following: From the dexcription of the problem, $g(x)$ should be non-decreasing. By construction, let $g(x)=\text{sup}_{r\leq x} g(r)$ to make sure $g(x)$ is non-decreasing. Then, how to do the following steps?

0
Q: For which values of $\alpha$ and $\beta$ does the integral $\int\limits_2^{\infty}\frac{dx}{x^{\alpha}ln^{\beta}x}$ converge?

Jose PaterninaI'm trying to find out for which values of $\alpha$ and $\beta$ does the integral $\int\limits_2^{\infty}\frac{dx}{x^{\alpha}ln^{\beta}x}$ converge. I know that when $\alpha=1$ then $\beta$ must be greater than $1$. I tried to use integration by parts but It didn't work, so I would appreciate so...

0
Q: Weak Solution to a PDE

Kevin DohertyThis is a problem out of Logan's Applied Math book. Section 6.7, problem 2. Show that for any locally integrable function f on $\mathbb{R}$ the function $u(x,y) = f(x-y)$ is a weak solution to the equation $u_x + u_y = 0$ on $\mathbb{R}^2$. I've got a solution attempt that expresses $f_x$ ...

0
Q: Integration problem in polar

MarussTHow to integrate double integral $$\int_{0}^{\infty}\!\int_{0}^{2\pi}\ \frac{1}{2}\left(\frac{\partial}{\partial x}-\frac{\partial}{\partial y}\right)g_m \bar{g_n} , d\theta dr$$ where $$g_a=(x+iy)^a$$ . I do not know how to differentiate first part of integral.

Short title. Title contains problem. Integration problem in polar
0
Q: Is this the correct way to compute the blow up of a curve

Lee WangI'm trying to calculate the blowup of the curve y^5=z^2-3z^3+2z^4 at (0,0) We have the relation Ay=Bz, now I split it into two charts: The first chart(y,a=A/B): y^5=a^2y^2-3a^3y^3+2a^4y^2-y^3=y^2(a^2-3a^3y+2a^4y^2-y^3) The second chart (x,b=B/A) 0=z^2-3z^3+2z^4+b^5z^3=z^2(1-3z+2z^2+zb^5) Now t...

This site uses MathJax formatting of formulas. More tips here. (autocomment)Normal Human 21 secs ago
0
Q: On an exercise that asks for a homeomorphism between a quotient space and a metrizable space.

MonoliteI have the solution to the exercise but have a doubt on one thing, I state the exercise: Given $$ X = \{ (x,y) \in R^2 | x = \frac{1}{n}, n \in N \}$$ and $Y = X/_{\sim}$ where the equivalence relation is $(x_1, y_1) \sim (x_2,y_2) \iff (x_1, y_1) = (x_2,y_2)$ or $ y_1 = y_2$ with $x_1,x_2 \in...

 
11:37 PM
0
Q: Genus-Degree formula gives the wrong answer

Lee WangI#m trying to compute the genus of the normalization of the curve: $y^5=x(x-1)(x-2)$ Now I calculate the ramification points of the projection x: they are $(0,0),(1,0),(2,0)$ and they are of ramification order 5, and possibly ramification points at the infinite point $(1:0:0)$. Now the curve is ...

0
Q: The value of the following limitation without using hopital

aliakbar$lim_{x\to 3}(\frac{x}{x-3}\int_{3}^{x}\frac{sint}{t})$ without using Hopital

0
Q: help needed to find the residues and contour integral

Xfrglkvfind the contour integral $$\oint _{c} \frac{\sinh z}{z-1} dz$$, where C is a square of side 3 centered at the origin I have problem both with finding the residues and doing the integral

Words such as help do not add information to titles. Please edit the title so that it better describes the specifics of your question. Do not hesitate to make it longer or include a formula if needed. More tips here. (from a bot)Normal Human 21 secs ago
0
Q: Ring homomorphisms.

Théodore RozencwajgI have a map α : $F$[x] -> $L$ , where $L$ is $F$[x]/(x²+2x-1) and $F$[x] is $Z$/5$Z$, and what it does is x -> t, where t is a square root of 2 in $L$. I need to prove it is a ring homomorphism. For that, I need to show first that α(1) = identity of L, then that α(a+b) = α(a) + α(b) and finally...

Short title. Ring homomorphisms.
0
Q: evaluating the limit $\lim \limits_{n \to \infty} \frac{(4(n*3^n + 3)^n}{(3^{n+1} (n+1)+3)^{n+1}}$

pokemonfanHi there im trying to solve the following limit $\lim \limits_{n \to \infty} \frac{(4(n*3^n + 3)^n}{(3^{n+1} (n+1)+3)^{n+1}}$, but I've got literally no idea where to even start, it's just too big! I don't know wether l'hospital would be a good idea considering, well, look at those functions. I a...

0
Q: Related Rates using circles

Shavana.Scan someone please help? I'm taking Calculus, but I'm really having trouble understanding the concept of related rates. A jogger runs around a circular track of radius 55 ft. Let (x,y) be her coordinates, where the origin is the center of the track. When the jogger's coordinates are (33, 44), he...

0
Q: A function which is a derivative of another function is continuous.

acegreSuppose $f$ is differentiable on $\mathbb{R}$ and its derivative $f'$ is continuous on the interval $[a,b]$. What constraints on $f$ would such condition give us?

0
Q: Linear feasibility problem of a system of totally unimodular inequalities

beeflavorI am given a system of linear inequalities $Ax \le b$ where $A$ is a $m$ by $n$ totally unimodular matrix. Let $A_1,\dots, A_m$ be the rows of $A$ and let $H_1,\dots, H_m$ be the hyperplanes in $\mathbb{R}^n$ corresponding to $A_ix = 0$. Suppose I know that the feasible region $P$ has the same di...

 
00:00 - 17:0017:00 - 00:00

« first day (41 days earlier)      last day (533 days later) »